User avatar
 
ManhattanPrepLSAT1
Thanks Received: 1909
Atticus Finch
Atticus Finch
 
Posts: 2851
Joined: October 07th, 2009
 
This post thanked 1 time.
 
 

Q15 - M: It is almost impossible

by ManhattanPrepLSAT1 Fri May 07, 2010 3:47 am

Let's think about the structure of this interesting question.

Speaking abstractly...

M's statement amounts to an observed phenomenon.

Q's statement offers an explanation for why the observed phenomenon is occurring.

Q's statement serves to undermine the use of the phenomenon by M to offer an alternative explanation.

Back into the words of the stimulus.

Most old people are right handed. Hmm. Why? Well maybe, right-handed people live longer. Or maybe left-handed people were taught to use their right hand for things like writing and eating. By Q offering the explanation that old people were taught to use their right hands for writing and eating, Q undermines the possible alternative explanation that right-handed people live longer. So answer choice (A) is one possible explanation of the statistic cited by M, but Q provides an alternative to this explanation. Thus, answer choice (A) is correct.

(A) is an alternative explanation undermined by Q's statement.
(B) is irrelevant. Social attitudes may or may not change, but this would not offer an alternative explanation for why so many old people are right-handed.
(C) is unsupported. There may have been harm caused by forcing people to switch handedness. Even if it were supported, it still wouldn't be correct. The correct answer should be an alternative explanation.
(D) might be tempting in that it relates handedness with genetic predispositions. This answer choice would have been better had it said that the genetic predisposition was for a longer life span.
(E) supports Q's statement but is not an alternative explanation to it.
 
zee.brad
Thanks Received: 3
Forum Guests
 
Posts: 28
Joined: February 02nd, 2012
 
 
 

Re: Q15 - M: It is almost impossible

by zee.brad Thu Sep 06, 2012 5:31 pm

Hi, I don't understand is why you stress "longer life span"? because M mentioned "85 and 90" in the phenomenon observed?, is it possible that they just want to state there is a certain period primarily right handed? And then Q gives ans to this phenomenon, which is many left handed were forced to use right hands during that period.

Then M and Q together, it implies that handiness is not solely genetic oriented? Some other factors, like social pressure can affect the handiness?
Which is exactly ans D

I am not so sure about M was trying to say "handiness and longer life span"

Thanks!
 
magnusgan
Thanks Received: 1
Jackie Chiles
Jackie Chiles
 
Posts: 42
Joined: March 25th, 2013
 
 
 

Re: Q15 - M: It is almost impossible

by magnusgan Sat May 04, 2013 4:00 pm

I couldn't understand the question (Q's response serves to...)

It was phrased in an unnecessarily complex manner! But after reading the posted explanation I was like oh so that's what they were asking.

Are questions phrased in such a manner common in the later LSATs?
 
economienda
Thanks Received: 1
Forum Guests
 
Posts: 21
Joined: June 12th, 2013
 
 
 

Re: Q15 - M: It is almost impossible

by economienda Fri Dec 12, 2014 1:27 am

The stem was hard to understand
 
courtney_chrusch
Thanks Received: 0
Vinny Gambini
Vinny Gambini
 
Posts: 7
Joined: March 03rd, 2016
 
 
 

Re: Q15 - M: It is almost impossible

by courtney_chrusch Sat Feb 04, 2017 5:21 pm

Hi,

Can someone please break this down? The previous responses have only furthered my confusion. Why is C and E wrong? What does this have anything to do with living longer? It just seems like a cohort thing to me. Please explain! So confused.
User avatar
 
ohthatpatrick
Thanks Received: 3808
Atticus Finch
Atticus Finch
 
Posts: 4661
Joined: April 01st, 2011
 
 
 

Re: Q15 - M: It is almost impossible

by ohthatpatrick Mon Feb 06, 2017 3:09 am

This question stem is indeed wretched. It's essentially saying this:

* M presented a stat about 85 to 90 year olds. We're gonna pretend it's his evidence (premise).

* The answer choices are each a conclusion M could draw.

* For the correct answer, Q's statement would weaken M's argument.

The question stem says that M would use its fact about 85-90 yr olds in support of the following answers. That means the answers are each a conclusion M could draw.

Q's statement is supposed to counter (weaken) M's hypothetical argument.

=====
STRONG LSAT SPIDEY-SENSE:
LSAT authors frequently present a statistic and then conclude "THIS is the reason for that statistic".

M presents a statistic, so it's probable that the conclusion M will draw (in the correct answer choice) will be of the form "THIS is the reason why you can't find a lefty in the 85-90 range".

Q's statement is an explanation for why you can't find a lefty in the 85-90 range, so if M's conclusion (the correct answer) poses a DIFFERENT explanation, then Q's statement will weaken it.
=======

If we don't have that suspicion, we'll basically just read each answer choice as though it's M's conclusion, and then ask ourselves, "Would Q's statement weaken this argument?"

(A) If M concluded this, he would be thinking that the being right handed makes you live longer, that's why you don't see any left handed people in the 85-90 range.

Q's statement would weaken that argument by saying, "No, buddy, the real reason you don't see lefties in the 85-90 yr old range is because that generation, while growing up, was FORCED to all do its basic skills right handed."

(B) If M said "you can find a lefty in the 85-90 year old range, therefore societal attitudes about handedness differ at different times", we'd have a head scratcher of an argument. How are THOSE even related? Furthermore, Q's statement sounds COMPATIBLE with (B), but we're looking for something that runs counter to where it's going.

(C) Similar to (B), this answer choice would be a bizarre conclusion for M to draw off of his 85-90 year old stat. And again, it's actually compatible with what Q was saying.

(D) Similar to (B) and (C) ... it doesn't make sense as a conclusion for M to draw on the basis of his evidence, and Q's statement seems to go WITH it, more than against it.

(E) Same as B/C/D in both regards.
 
aaronwfrank
Thanks Received: 2
Vinny Gambini
Vinny Gambini
 
Posts: 23
Joined: August 24th, 2016
 
 
 

Re: Q15 - M: It is almost impossible

by aaronwfrank Tue Feb 14, 2017 6:32 pm

For anyone still having trouble discounting answer D, the answer is wrong because "social pressures" is actually supported by Q's response, whereas "genetic disposition" is the part being denied support. Only half of it is correct. In having "social pressures" being supported by Q, we have a violation of the rule set out in the stem, which says "ANY use by M of Q's evidence...in support of which of the following hypotheses." With answer A, however, the support for conferring a survival advantage, is completely denied, since any outside influences or alternate explanations are dismissed.